Difference between revisions of "2013 AMC 12A Problems/Problem 19"
(→Solution 1 (Number theoretic power of a point)) |
(→Solution 1 (Number theoretic power of a point)) |
||
Line 30: | Line 30: | ||
</asy> | </asy> | ||
− | Let <math> | + | Let circle <math>A</math> intersect <math>AC</math> at <math>D</math> and <math>E</math> as shown. We apply Power of a Point on point <math>C</math> with respect to circle <math>A.</math> |
− | |||
− | + | We have | |
− | + | \begin{align*} | |
− | + | CX \cdot CB &= CD \cdot C \\ | |
+ | CX(CX+XB) &= (97-86)(97+86) \\ | ||
+ | CX(CX+XB) &= 3 \cdot 11 \cdot 61. | ||
+ | \end{align*} | ||
− | + | Since lengths cannot be negative, we must have <math>CX+XB \ge CX.</math> This generates the four solution pairs <cmath>(CX,XB)=(1,2013) \qquad (3,671) \qquad (11,183) \qquad (33,61).</cmath> | |
− | + | However, by the Triangle Inequality on <math>\triangle ACX,</math> we see that <math>CX>13.</math> This implies that we must have <math>CX+XB= \boxed{\textbf{(D) }61}.</math> (Solution by unknown, latex/asy modified majorly by samrocksnature) | |
− | |||
− | |||
===Solution 2=== | ===Solution 2=== |
Revision as of 20:07, 1 July 2021
Contents
Problem
In , , and . A circle with center and radius intersects at points and . Moreover and have integer lengths. What is ?
Solution
Solution 1 (Number theoretic power of a point)
Let circle intersect at and as shown. We apply Power of a Point on point with respect to circle
We have \begin{align*} CX \cdot CB &= CD \cdot C \\ CX(CX+XB) &= (97-86)(97+86) \\ CX(CX+XB) &= 3 \cdot 11 \cdot 61. \end{align*}
Since lengths cannot be negative, we must have This generates the four solution pairs However, by the Triangle Inequality on we see that This implies that we must have (Solution by unknown, latex/asy modified majorly by samrocksnature)
Solution 2
Let , , and meet the circle at and , with on . Then . Using the Power of a Point, we get that . We know that , and that by the triangle inequality on . Thus, we get that
Solution 3
Let represent , and let represent . Since the circle goes through and , . Then by Stewart's Theorem,
(Since cannot be equal to , dividing both sides of the equation by is allowed.)
The prime factors of are , , and . Obviously, . In addition, by the Triangle Inequality, , so . Therefore, must equal , and must equal
Video Solution by Richard Rusczyk
https://artofproblemsolving.com/videos/amc/2013amc12a/357
~dolphin7
Video Solution
~sugar_rush
See also
2013 AMC 12A (Problems • Answer Key • Resources) | |
Preceded by Problem 18 |
Followed by Problem 20 |
1 • 2 • 3 • 4 • 5 • 6 • 7 • 8 • 9 • 10 • 11 • 12 • 13 • 14 • 15 • 16 • 17 • 18 • 19 • 20 • 21 • 22 • 23 • 24 • 25 | |
All AMC 12 Problems and Solutions |
The problems on this page are copyrighted by the Mathematical Association of America's American Mathematics Competitions.